Inscription / Connexion Nouveau Sujet
Niveau Maths sup
Partager :

Somme d'Arctan

Posté par
Torto
30-09-09 à 15:39

Bonjour. Je suis bloqué face à une suite dont on me demande sa convergence et sa limite.

  sn = Arctan( 1/ (p²+p+1) )   pour p variant de 0 à n.

J'ai d'abord essayer de simplifier l'expression, j'ai aboutit à:  (n+1)/2 - Arctan( p²+p+1 ) pour p variant de 0 à n.

Puis j'ai calculé: sn+1-sn= /2 - Arctan((n+1)(n+2))

Et là, je ne sais trop quoi faire...
Merci de votre aide!

Posté par
perroquet
re : Somme d'Arctan 30-09-09 à 15:43

Bonjour, Torto

Il suffit de remarquer:
3$ {\rm Arctan}(n+1)-{\rm Arctan} (n)={\rm Arctan}\left(\frac{1}{n^2+n+1}\right)

Posté par
Torto
re : Somme d'Arctan 30-09-09 à 17:49

Ah oui, effectivement avec la formule tan(a-b)....

On trouve que sn=Arctan(n+1)

Donc sa limite en l'infini est 1, et elle est convergente.  

Merci encore!

Posté par
perroquet
re : Somme d'Arctan 30-09-09 à 18:20

Sa limite en l'infini est égale à  /2  (petite étourderie )

Posté par
Torto
re : Somme d'Arctan 30-09-09 à 19:49

Oui... J'ai parlé dans l'élan de la réponse!
Merci encore



Vous devez être membre accéder à ce service...

Pas encore inscrit ?

1 compte par personne, multi-compte interdit !

Ou identifiez-vous :


Rester sur la page

Inscription gratuite

Fiches en rapport

parmi 1675 fiches de maths

Désolé, votre version d'Internet Explorer est plus que périmée ! Merci de le mettre à jour ou de télécharger Firefox ou Google Chrome pour utiliser le site. Votre ordinateur vous remerciera !